2008 AMC 8 Problems/Problem 12

Revision as of 17:18, 8 August 2021 by Raina0708 (talk | contribs) (Solution)

Problem

A ball is dropped from a height of $3$ meters. On its first bounce it rises to a height of $2$ meters. It keeps falling and bouncing to $\frac{2}{3}$ of the height it reached in the previous bounce. On which bounce will it not rise to a height of $0.5$ meters?

$\textbf{(A)}\  3 \qquad \textbf{(B)}\  4 \qquad \textbf{(C)}\ 5 \qquad \textbf{(D)}\ 6 \qquad \textbf{(E)}\ 7$

Solution

Each bounce is 2/3 times the height of the previous bounce the first bounce reaches 2 meters, the second 4/3 the third 8/9 the fourth 16/27 and the fifth 32/81 Half of 81 is 40.5 so the ball does not reach the required height on bounce C:5

See Also

2008 AMC 8 (ProblemsAnswer KeyResources)
Preceded by
Problem 11
Followed by
Problem 13
1 2 3 4 5 6 7 8 9 10 11 12 13 14 15 16 17 18 19 20 21 22 23 24 25
All AJHSME/AMC 8 Problems and Solutions

The problems on this page are copyrighted by the Mathematical Association of America's American Mathematics Competitions. AMC logo.png